LSAT and Law School Admissions Forum

Get expert LSAT preparation and law school admissions advice from PowerScore Test Preparation.

 RMahmood
  • Posts: 2
  • Joined: Apr 03, 2013
|
#8878
Question number 3 I had originally selected C. While I see how the answer is A and that makes sense to me....wouldn't L being 6 make M automatically 3rd?

I understand nothing can come after L if it is last then it could violate the L>O rule...

from my understanding is I am taking the wrong rule and applying it. but then again if L is last then M must be 3rd? which would negate the other rule entirely not having to worry about L>O.
User avatar
 Dave Killoran
PowerScore Staff
  • PowerScore Staff
  • Posts: 5853
  • Joined: Mar 25, 2011
|
#8891
Hi R,

Thanks for the question. You are making a Mistaken Reversal with the first rule. Just because L is sixth doesn't mean M must be third, it just means M can be third. Answer choice (C) is there primarily to catch people who make this mistake (which is ok--one reason that preparing for the LSAT is useful is because you can see traps like this and can avoid them next time!).

Please let me know if that helps. Thanks!
 RMahmood
  • Posts: 2
  • Joined: Apr 03, 2013
|
#8892
:-D

Thanks that makes sense that I should not make the assumption the other way around for L and M. I will try to keep that in mind for future questions! A very easy trap I can see why I feel for answer choice C.

Thank Dave.

Get the most out of your LSAT Prep Plus subscription.

Analyze and track your performance with our Testing and Analytics Package.